Ορθογώνιο τρίγωνο και ευκαιρία για λύσεις!

Συντονιστής: chris_gatos

Άβαταρ μέλους
chris_gatos
Επιμελητής
Δημοσιεύσεις: 6962
Εγγραφή: Κυρ Δεκ 21, 2008 9:03 pm
Τοποθεσία: Ανθούπολη

Ορθογώνιο τρίγωνο και ευκαιρία για λύσεις!

#1

Μη αναγνωσμένη δημοσίευση από chris_gatos » Τετ Απρ 01, 2020 10:33 pm

Δίνεται ένα ορθογώνιο τρίγωνο AB\Gamma με υποτείνουσα την B\Gamma.
Πάνω στην υποτείνουσα παίρνουμε τα σημεία M και N,
ώστε να ισχύει \Gamma M=\Gamma A
και BN=BA.
Να αποδείξετε ότι η γωνία M\widehat{A}N είναι ίση με 45^{\circ}.


Χρήστος Κυριαζής

Λέξεις Κλειδιά:
Μιχάλης Τσουρακάκης
Δημοσιεύσεις: 2769
Εγγραφή: Παρ Ιαν 11, 2013 4:17 am
Τοποθεσία: Ηράκλειο Κρήτης

Re: Ορθογώνιο τρίγωνο και ευκαιρία για λύσεις!

#2

Μη αναγνωσμένη δημοσίευση από Μιχάλης Τσουρακάκης » Τετ Απρ 01, 2020 10:55 pm

chris_gatos έγραψε:
Τετ Απρ 01, 2020 10:33 pm
Δίνεται ένα ορθογώνιο τρίγωνο AB\Gamma με υποτείνουσα την B\Gamma.
Πάνω στην υποτείνουσα παίρνουμε τα σημεία M και N,
ώστε να ισχύει \Gamma M=\Gamma A
και BN=BA.
Να αποδείξετε ότι η γωνία M\widehat{A}N είναι ίση με 45^{\circ}.
AB,AC είναι εφαπτόμενες των κύκλων (C,CA),(B,BA) αντίστοιχα.

Άρα x= \dfrac{C}{2} ,y= \dfrac{B}{2 } \Rightarrow x+y= \dfrac{B+C}{2}= \dfrac{90^0}{2}=45^0 \Rightarrow  \angle MAN=45^0
45.png
45.png (12.89 KiB) Προβλήθηκε 1512 φορές


Λάμπρος Κατσάπας
Δημοσιεύσεις: 838
Εγγραφή: Σάβ Ιουν 17, 2017 10:17 pm
Τοποθεσία: Αθήνα

Re: Ορθογώνιο τρίγωνο και ευκαιρία για λύσεις!

#3

Μη αναγνωσμένη δημοσίευση από Λάμπρος Κατσάπας » Τετ Απρ 01, 2020 10:57 pm

chris_gatos έγραψε:
Τετ Απρ 01, 2020 10:33 pm
Δίνεται ένα ορθογώνιο τρίγωνο AB\Gamma με υποτείνουσα την B\Gamma.
Πάνω στην υποτείνουσα παίρνουμε τα σημεία M και N,
ώστε να ισχύει \Gamma M=\Gamma A
και BN=BA.
Να αποδείξετε ότι η γωνία M\widehat{A}N είναι ίση με 45^{\circ}.
Φέρνουμε τους κύκλους (B,BA),(C,CA).

CA,BA είναι εφαπτομένες στους κύκλους (B,BA),(C,CA) αντίστοιχα

αφού \angle BAC=90^{\circ}. Άρα \angle NAC=\angle ABC/2,\angle MAB=\angle ACB/2

(γωνία χορδής εφαπτομένης και επίκεντρη που βαίνουν στο ίδιο τόξο).

Προσθέτωντας κατά μέλη τις τελευταίες παίρνουμε

\angle BAM+\angle NAC=45^{\circ} και επομένως \angle MAN=45^{\circ}.


Άβαταρ μέλους
Γιώργος Μήτσιος
Δημοσιεύσεις: 1789
Εγγραφή: Κυρ Ιούλ 01, 2012 10:14 am
Τοποθεσία: Aρτα

Re: Ορθογώνιο τρίγωνο και ευκαιρία για λύσεις!

#4

Μη αναγνωσμένη δημοσίευση από Γιώργος Μήτσιος » Τετ Απρ 01, 2020 11:29 pm

Καλό βράδυ!
Ορθογώνιο τρίγωνο.PNG
Ορθογώνιο τρίγωνο.PNG (6.82 KiB) Προβλήθηκε 1487 φορές
Από τα ισοσκελή τρίγωνα BAN,CAM παίρνουμε \widehat{AMC}=90^\circ -\widehat{C}/2 και \widehat{ANB}=90^\circ -\widehat{B}/2.

Τότε στο τρίγωνο MAN έχουμε \widehat{M}+\widehat{N}=180^\circ -\dfrac{\widehat{B}+\widehat{C}}{2}=135^\circ άρα \widehat{MAN}=45^\circ.
Φιλικά, Γιώργος.


Άβαταρ μέλους
george visvikis
Επιμελητής
Δημοσιεύσεις: 13272
Εγγραφή: Παρ Νοέμ 01, 2013 9:35 am

Re: Ορθογώνιο τρίγωνο και ευκαιρία για λύσεις!

#5

Μη αναγνωσμένη δημοσίευση από george visvikis » Πέμ Απρ 02, 2020 9:15 am

chris_gatos έγραψε:
Τετ Απρ 01, 2020 10:33 pm
Δίνεται ένα ορθογώνιο τρίγωνο AB\Gamma με υποτείνουσα την B\Gamma.
Πάνω στην υποτείνουσα παίρνουμε τα σημεία M και N,
ώστε να ισχύει \Gamma M=\Gamma A
και BN=BA.
Να αποδείξετε ότι η γωνία M\widehat{A}N είναι ίση με 45^{\circ}.
Ορθογ. τρίγωνο.png
Ορθογ. τρίγωνο.png (16.6 KiB) Προβλήθηκε 1442 φορές
Τα ύψη BL, CK των ισοσκελών τριγώνων BAN, CAM είναι και διχοτόμοι, οπότε τέμνονται στο έγκεντρο I του

τριγώνου ABC. Άρα το AKIL είναι εγγράψιμο κι επειδή \displaystyle B\widehat IC = 90^\circ  + \frac{{\widehat A}}{2} = 135^\circ  \Rightarrow \boxed{M\widehat AN=45^\circ}


Άβαταρ μέλους
apotin
Δημοσιεύσεις: 845
Εγγραφή: Τετ Απρ 08, 2009 5:53 pm

Re: Ορθογώνιο τρίγωνο και ευκαιρία για λύσεις!

#6

Μη αναγνωσμένη δημοσίευση από apotin » Πέμ Απρ 02, 2020 10:10 am

chris_gatos έγραψε:
Τετ Απρ 01, 2020 10:33 pm
Δίνεται ένα ορθογώνιο τρίγωνο AB\Gamma με υποτείνουσα την B\Gamma.
Πάνω στην υποτείνουσα παίρνουμε τα σημεία M και N,
ώστε να ισχύει \Gamma M=\Gamma A
και BN=BA.
Να αποδείξετε ότι η γωνία M\widehat{A}N είναι ίση με 45^{\circ}.
g45.png
g45.png (23.78 KiB) Προβλήθηκε 1431 φορές
Από το ισοσκελές τρίγωνο \displaystyle{AE\Gamma } έχουμε

\displaystyle{\hat E = 90^\circ  - {{\hat \Gamma } \over 2}}

Από το ισοσκελές τρίγωνο \displaystyle{AB\Delta } έχουμε

\displaystyle{\hat \Delta  = 90^\circ  - {{\hat B} \over 2}}

Άρα

\displaystyle{\hat E + \hat \Delta  = 180^\circ  - {{\hat B + \hat \Gamma } \over 2} = 135^\circ }

Οπότε από το τρίγωνο \displaystyle{A\Delta E} έχουμε

\displaystyle{\Delta \hat AE = 180^\circ  - \left( {\hat E + \hat \Delta } \right) = 45^\circ }

Edit: Τώρα πρόσεξα πως η λύση μου είναι ίδια με του Γιώργου Μήτσιου. Βλέποντας τα τόξα στο σχήμα του υπέθεσα ότι ήταν διαφορετική.
τελευταία επεξεργασία από apotin σε Πέμ Απρ 02, 2020 7:20 pm, έχει επεξεργασθεί 1 φορά συνολικά.


Αποστόλης
Άβαταρ μέλους
KARKAR
Δημοσιεύσεις: 15012
Εγγραφή: Τετ Δεκ 08, 2010 6:18 pm

Re: Ορθογώνιο τρίγωνο και ευκαιρία για λύσεις!

#7

Μη αναγνωσμένη δημοσίευση από KARKAR » Πέμ Απρ 02, 2020 1:38 pm

Ευκαιρία.png
Ευκαιρία.png (10.36 KiB) Προβλήθηκε 1396 φορές
\phi+\theta=90^0-\dfrac{\omega}{2} , \phi+\eta=45^0+\dfrac{\omega}{2} , οπότε με πρόσθεση :

2\phi+\theta+\eta=135^0 , άρα : \phi=45^0 , αφού : \theta+\phi+\eta=90^0


p_gianno
Δημοσιεύσεις: 1084
Εγγραφή: Κυρ Δεκ 21, 2008 1:10 am

Re: Ορθογώνιο τρίγωνο και ευκαιρία για λύσεις!

#8

Μη αναγνωσμένη δημοσίευση από p_gianno » Πέμ Απρ 02, 2020 4:43 pm

45 moires.png
45 moires.png (21.31 KiB) Προβλήθηκε 1368 φορές

Στο ισοσκελές τργ ACM φέρω τα ύψη CI, AD. Τότε \angle A_1 =\angle C_1=\angle C/2

Ομοίως \angle A_2 =\angle B/2 . \,\,\,\, Συνεπώς

\angle MAN=\angle A_1+\angle A_2=\angle(B+C)/2=90^0/2=45^0


Άβαταρ μέλους
Γιώργος Μήτσιος
Δημοσιεύσεις: 1789
Εγγραφή: Κυρ Ιούλ 01, 2012 10:14 am
Τοποθεσία: Aρτα

Re: Ορθογώνιο τρίγωνο και ευκαιρία για λύσεις!

#9

Μη αναγνωσμένη δημοσίευση από Γιώργος Μήτσιος » Πέμ Απρ 02, 2020 8:26 pm

Καλησπέρα. Μια παραλλαγή - κάπως ..μακρύτερη- για την ποικιλία.
Ορθ. τρίγωνο ΙΙ.PNG
Ορθ. τρίγωνο ΙΙ.PNG (10.02 KiB) Προβλήθηκε 1341 φορές
Φέρω AP \perp AM με P \in BC. Στο ορθογώνιο τρίγωνο PAM είναι AC=MC άρα η AC διάμεσος κι' έτσι \widehat{APC}=\dfrac{\widehat{C}}{2}.

Τότε \widehat{PAN}=\dfrac{\pi -B}{2}-\dfrac{C}{2}=\dfrac{\pi }{4} και συνεπώς \widehat{MAN}=\dfrac{\pi }{4}. Φιλικά Γιώργος.


Άβαταρ μέλους
Doloros
Επιμελητής
Δημοσιεύσεις: 9848
Εγγραφή: Τρί Αύγ 07, 2012 4:09 am
Τοποθεσία: Ιεράπετρα Κρήτης

Re: Ορθογώνιο τρίγωνο και ευκαιρία για λύσεις!

#10

Μη αναγνωσμένη δημοσίευση από Doloros » Παρ Απρ 03, 2020 8:57 pm

chris_gatos έγραψε:
Τετ Απρ 01, 2020 10:33 pm
Δίνεται ένα ορθογώνιο τρίγωνο AB\Gamma με υποτείνουσα την B\Gamma.
Πάνω στην υποτείνουσα παίρνουμε τα σημεία M και N,
ώστε να ισχύει \Gamma M=\Gamma A
και BN=BA.
Να αποδείξετε ότι η γωνία M\widehat{A}N είναι ίση με 45^{\circ}.
Ορθογώνιο τρίγωνο και ευκαιρία για λύσεις.png
Ορθογώνιο τρίγωνο και ευκαιρία για λύσεις.png (21.32 KiB) Προβλήθηκε 1291 φορές
Έστω D το σημείο τομής της AC με την από το N παράλληλη στην MA.

Προφανώς το τρίγωνο CDN είναι ισοσκελές με κορυφή το C και \widehat {{x_{}}} = \widehat {{y_{}}}

Από τα ισοσκελή τρίγωνα : BNA\,\,\kappa \alpha \iota \,\,CDN έχω :

\left\{ \begin{gathered} 
  2\widehat {{\theta _{}}} + \widehat {{B_{}}} = 180^\circ  \hfill \\ 
  2\widehat {{\omega _{}}} + \widehat {{C_{}}} = 180^\circ  \hfill \\  
\end{gathered}  \right. \Rightarrow \widehat {{\theta _{}}} + \widehat {{\omega _{}}} + \dfrac{{\widehat {{B_{}}} + \widehat {{C_{}}}}}{2} = 180^\circ  \Rightarrow \widehat {{\theta _{}}} + \widehat {{\omega _{}}} = 135^\circ

Αλλά \widehat {{\theta _{}}} + \widehat {{y_{}}} + \widehat {{\omega _{}}} = 180^\circ  \Rightarrow \widehat {{y_{}}} = 45^\circ  \Rightarrow \widehat {{x_{}}} = 45^\circ


Άβαταρ μέλους
Γιώργος Ρίζος
Επιμελητής
Δημοσιεύσεις: 5283
Εγγραφή: Δευ Δεκ 29, 2008 1:18 pm
Τοποθεσία: Κέρκυρα

Re: Ορθογώνιο τρίγωνο και ευκαιρία για λύσεις!

#11

Μη αναγνωσμένη δημοσίευση από Γιώργος Ρίζος » Παρ Απρ 03, 2020 11:27 pm

Kαλησπέρα σε όλους.

03-04-2020 Γεωμετρία.png
03-04-2020 Γεωμετρία.png (35.41 KiB) Προβλήθηκε 1268 φορές

Eίναι  \displaystyle \widehat {ANM} = \widehat {BAD} \Leftrightarrow \frac{{\mathop {AB}\limits^ \cap   + \mathop {CD}\limits^ \cap  }}{2} = \frac{{\mathop {BE}\limits^ \cap   + \mathop {ED}\limits^ \cap  }}{2}

και  \displaystyle \widehat {AMN} = \widehat {CAE} \Leftrightarrow \frac{{\mathop {AC}\limits^ \cap   + \mathop {BE}\limits^ \cap  }}{2} = \frac{{\mathop {CD}\limits^ \cap   + \mathop {ED}\limits^ \cap  }}{2}


Οπότε, προσθέτοντας κατά μέλη είναι

 \displaystyle \frac{{\mathop {AB}\limits^ \cap   + \mathop {CD}\limits^ \cap  }}{2} + \frac{{\mathop {AC}\limits^ \cap   + \mathop {BE}\limits^ \cap  }}{2} = \frac{{\mathop {CD}\limits^ \cap   + \mathop {ED}\limits^ \cap  }}{2} + \frac{{\mathop {BE}\limits^ \cap   + \mathop {ED}\limits^ \cap  }}{2} \Leftrightarrow \frac{{\mathop {AB}\limits^ \cap   + \mathop {AC}\limits^ \cap  }}{2} = \mathop {ED}\limits^ \cap

άρα  \displaystyle \mathop {ED}\limits^ \cap   = \frac{{180^\circ }}{2} = 90^\circ  \Leftrightarrow \widehat {MAN} = 45^\circ .


Μιχάλης Τσουρακάκης
Δημοσιεύσεις: 2769
Εγγραφή: Παρ Ιαν 11, 2013 4:17 am
Τοποθεσία: Ηράκλειο Κρήτης

Re: Ορθογώνιο τρίγωνο και ευκαιρία για λύσεις!

#12

Μη αναγνωσμένη δημοσίευση από Μιχάλης Τσουρακάκης » Σάβ Απρ 04, 2020 3:06 am

chris_gatos έγραψε:
Τετ Απρ 01, 2020 10:33 pm
Δίνεται ένα ορθογώνιο τρίγωνο AB\Gamma με υποτείνουσα την B\Gamma.
Πάνω στην υποτείνουσα παίρνουμε τα σημεία M και N,
ώστε να ισχύει \Gamma M=\Gamma A
και BN=BA.
Να αποδείξετε ότι η γωνία M\widehat{A}N είναι ίση με 45^{\circ}.
Θεωρούμε τον κύκλο  (A,M,N)

Είναι \angle PAN= \angle ANM και  \angle MAQ= \angle NMA άρα PANM,AMNQ ισοσκελή

τραπέζια και οι πράσινες γωνίες είναι ίσες

Επειδή PQ διάμετρος, 2 \theta =90^0 \Rightarrow  \theta =45^0
γωνία.png
γωνία.png (12.88 KiB) Προβλήθηκε 1240 φορές


Απάντηση

Επιστροφή σε “Προτεινόμενα Θέματα Μαθηματικών”

Μέλη σε σύνδεση

Μέλη σε αυτήν τη Δ. Συζήτηση: Δεν υπάρχουν εγγεγραμμένα μέλη και 5 επισκέπτες